Chức năng tính toán hiệu quả như một ví dụ ngược với phỏng đoán Mobius của Sarnak


35

Gần đây Gil KalaiDick Lipton đều viết một bài báo hay về một phỏng đoán thú vị được đề xuất bởi Peter Sarnak, một chuyên gia về lý thuyết số và Giả thuyết Riemann.

Phỏng đoán. Đặt là hàm Möbius . Giả sử là một hàm với đầu vào dưới dạng biểu diễn nhị phân của , sau đó f : N{ - 1 , 1 } Một C 0μ(k)f:N{-1,1}MộtC0k Σ k n μ ( k ) f ( k ) = o ( n ) .kk

Σknμ(k)f(k)= =o(n).

Lưu ý rằng nếu thì chúng ta có một dạng tương đương của định lý số Prime .f(k)= =1

CẬP NHẬT : Ben Green trên MathOverflow cung cấp một bài viết ngắn tuyên bố để chứng minh phỏng đoán. Hãy nhìn vào tờ giấy .

Mặt khác, chúng tôi biết rằng bằng cách đặt (với một chút sửa đổi để phạm vi nằm trong ), tổng kết quả có ước tính Có một giới hạn trên rằng có thể được tính trong , do đó, giới hạn được đề xuất trên trong phỏng đoán không thể được nới lỏng thành hàm . Câu hỏi của tôi là:f(k)= =μ(k)-1,1μ(k)UPcoUPNPcoNPf(k)NP

Σknμ(k)2= =Ω(n).
μ(k)UPcoBạnPNPcoNPf(k)NP

Lớp có độ phức tạp thấp nhất mà chúng ta hiện biết là gì, sao cho hàm trong thỏa mãn ước lượng Đặc biệt, kể từ khi một số nhà lý luận cho rằng máy tính không có trong , chúng tôi có thể cung cấp khác chức năng trong đó ngụ ý một sự tăng trưởng tuyến tính trong tổng kết? Giới hạn thậm chí tốt hơn có thể thu được?Cf(k)C

Σknμ(k)f(k)= =Ω(n)?
μ(k)PPf(k)

3
Một số lớp lượng tử như P ^ {BQNC} cũng sẽ hoạt động, vì bao thanh toán nằm trong lớp đó.
Robin Kothari

5
Điều này thậm chí còn được biết nếu f(k)= =ktôi cho một cố định ? tôi
Manu

2
@Emanuele, câu hỏi hay. Hàm chỉ thị của bit thứ i trong biểu diễn nhị phân của k là "đa thức khung" tuyến tính, nhưng nó có hệ số rất cao, do đó, nó có thể không tuân theo định lý Green-Tao về mối tương quan của hàm Mobius với giới hạn -một hậu quả. Các kết quả không giới hạn bước có các đa thức khung giới hạn là các trường hợp đặc biệt, nhưng kết quả của chúng có thể có một số hạn chế về độ lớn của các hệ số
Luca Trevisan

1
Và nó được biết đến với ? fNC0
vật

Bạn có muốn một hàm với phạm vi { - 1 , 0 , 1 } hoặc { - 1 , 1 } hay cái gì khác không?f{-1,0,1}{-1,1}
Jukka Suomela

Câu trả lời:


4

Đã có những phát triển thú vị về vấn đề này, tuy nhiên việc thay thế bằng ACC (2) (Cụ thể là cũng cho phép cổng mod 2) vẫn nằm ngoài tầm với. Một số tiến bộ ngoài định lý của Ben Green có thể được tìm thấy trong câu hỏi MO này https://mathoverflow.net/questions/57543/walsh-fourier-transform-of-the-mobius-feft cũng như câu hỏi nàyMộtC0 https://mathoverflow.net / câu hỏi / 97261 / mobius-ngẫu nhiên-của-rudin-shapiro-trình tự . Ngoài ra, Jean Bourgain đã chứng minh tính ngẫu nhiên của Mobius cho mọi hàm đơn điệu (về mặt mở rộng chữ số nhị phân).f

Khi sử dụng trang web của chúng tôi, bạn xác nhận rằng bạn đã đọc và hiểu Chính sách cookieChính sách bảo mật của chúng tôi.
Licensed under cc by-sa 3.0 with attribution required.